Calculus, 10th Edition (Anton)

Published by Wiley
ISBN 10: 0-47064-772-8
ISBN 13: 978-0-47064-772-1

Chapter 9 - Infinite Series - 9.2 Monotone Sequences - Exercises Set 9.2 - Page 614: 32

Answer

(a) We obtain the following: $\mathop \smallint \limits_1^n \ln x{\rm{d}}x \lt \ln n! \lt \mathop \smallint \limits_1^{n + 1} \ln x{\rm{d}}x$ (b) We show that $\dfrac{{{n^n}}}{{{{\rm{e}}^{n - 1}}}} \lt n! \lt \dfrac{{{{\left( {n + 1} \right)}^{n + 1}}}}{{{{\rm{e}}^n}}}$, ${\ \ \ }$ for $n \gt 1$ (c) We prove that $\mathop {\lim }\limits_{n \to + \infty } \dfrac{{\sqrt[n]{{n!}}}}{n} = \dfrac{1}{{\rm{e}}}$

Work Step by Step

(a) 1. Referring to the graph on the left of Figure Ex-32, we see that the width of the subintervals is $\Delta n = 1$. Thus, the area of the rectangles is ${\rm{Area1}} = \ln 1 + \ln 2 + \ln 3 + ... + \ln \left( {n - 1} \right)$ ${\rm{Area1}} = \ln \left( {2\cdot3\cdot ... \cdot\left( {n - 1} \right)} \right) = \ln \left( {\left( {n - 1} \right)!} \right)$ The graph depicts the fact that $\mathop \smallint \limits_1^n \ln x{\rm{d}}x \lt {\rm{Area1}}$ Or $\mathop \smallint \limits_1^n \ln x{\rm{d}}x \lt \ln \left( {\left( {n - 1} \right)!} \right)$ Since $\ln \left( {\left( {n - 1} \right)!} \right) \lt \ln n!$, consequently (1) ${\ \ \ \ \ \ \ }$ $\mathop \smallint \limits_1^n \ln x{\rm{d}}x \lt \ln n!$ 2. Referring to the graph on the right of Figure Ex-32, we also see that the width of the subintervals is $\Delta n = 1$. Thus, the area of the rectangles is ${\rm{Area2}} = \ln 2 + \ln 3 + \ln 4 + ... + \ln n$ ${\rm{Area2}} = \ln \left( {2\cdot3\cdot4\cdot ...\cdot n} \right) = \ln n!$ The graph depicts the fact that $\mathop \smallint \limits_1^{n + 1} \ln x{\rm{d}}x \gt {\rm{Area2}}$ Or (2) ${\ \ \ \ \ \ \ }$ $\mathop \smallint \limits_1^{n + 1} \ln x{\rm{d}}x \gt \ln n!$ Combining equation (1) and equation (2), we obtain (3) ${\ \ \ \ \ \ \ }$ $\mathop \smallint \limits_1^n \ln x{\rm{d}}x \lt \ln n! \lt \mathop \smallint \limits_1^{n + 1} \ln x{\rm{d}}x$ (b) Evaluate the integrals: From the Table of Integrals, we know that $\mathop \smallint \limits_{}^{} \ln u{\rm{d}}u = u\ln u - u + C$ So, $\mathop \smallint \limits_1^n \ln x{\rm{d}}x = n\ln n - n + 1 = 1 - n + \ln {n^n}$ $\mathop \smallint \limits_1^{n + 1} \ln x{\rm{d}}x = \left( {n + 1} \right)\ln \left( {n + 1} \right) - n = - n + \ln {\left( {n + 1} \right)^{n + 1}}$ Substituting these results into equation (3) gives $1 - n + \ln {n^n} \lt \ln n! \lt - n + \ln {\left( {n + 1} \right)^{n + 1}}$ Taking the exponentials, we get ${{\rm{e}}^{1 - n + \ln {n^n}}} \lt {{\rm{e}}^{\ln n!}} \lt {{\rm{e}}^{ - n + \ln {{\left( {n + 1} \right)}^{n + 1}}}}$ $\dfrac{{{{\rm{e}}^{\ln {n^n}}}}}{{{{\rm{e}}^{n - 1}}}} \lt {{\rm{e}}^{\ln n!}} \lt \dfrac{{{{\rm{e}}^{\ln {{\left( {n + 1} \right)}^{n + 1}}}}}}{{{{\rm{e}}^n}}}$ Since ${{\rm{e}}^{\ln x}} = x$, so $\dfrac{{{n^n}}}{{{{\rm{e}}^{n - 1}}}} \lt n! \lt \dfrac{{{{\left( {n + 1} \right)}^{n + 1}}}}{{{{\rm{e}}^n}}}$ Hence, $\dfrac{{{n^n}}}{{{{\rm{e}}^{n - 1}}}} \lt n! \lt \dfrac{{{{\left( {n + 1} \right)}^{n + 1}}}}{{{{\rm{e}}^n}}}$, ${\ \ \ }$ for $n \gt 1$ (c) From part (b), we obtain $\dfrac{{{n^n}}}{{{{\rm{e}}^{n - 1}}}} \lt n! \lt \dfrac{{{{\left( {n + 1} \right)}^{n + 1}}}}{{{{\rm{e}}^n}}}$ Or $\dfrac{{{\rm{e}}{n^n}}}{{{{\rm{e}}^n}}} \lt n! \lt \dfrac{{{{\left( {n + 1} \right)}^{n + 1}}}}{{{{\rm{e}}^n}}}$ Take the $n$th root: ${\left( {\dfrac{{{\rm{e}}{n^n}}}{{{{\rm{e}}^n}}}} \right)^{1/n}} \lt \sqrt[n]{{n!}} \lt {\left( {\dfrac{{{{\left( {n + 1} \right)}^{n + 1}}}}{{{{\rm{e}}^n}}}} \right)^{1/n}}$ $\dfrac{{n{{\rm{e}}^{1/n}}}}{{\rm{e}}} \lt \sqrt[n]{{n!}} \lt \dfrac{{{{\left( {n + 1} \right)}^{1 + \dfrac{1}{n}}}}}{{\rm{e}}}$ $\dfrac{{n{{\rm{e}}^{1/n}}}}{{\rm{e}}} \lt \sqrt[n]{{n!}} \lt \dfrac{{\left( {n + 1} \right){{\left( {n + 1} \right)}^{\dfrac{1}{n}}}}}{{\rm{e}}}$ Divide by $n$: $\dfrac{{{{\rm{e}}^{1/n}}}}{{\rm{e}}} \lt \dfrac{{\sqrt[n]{{n!}}}}{n} \lt \dfrac{{\left( {1 + \dfrac{1}{n}} \right){{\left( {n + 1} \right)}^{\dfrac{1}{n}}}}}{{\rm{e}}}$ Take the limits: $\mathop {\lim }\limits_{n \to \infty } \dfrac{{{{\rm{e}}^{1/n}}}}{{\rm{e}}} \lt \mathop {\lim }\limits_{n \to \infty } \dfrac{{\sqrt[n]{{n!}}}}{n} \lt \mathop {\lim }\limits_{n \to \infty } \dfrac{{\left( {1 + \dfrac{1}{n}} \right){{\left( {n + 1} \right)}^{\dfrac{1}{n}}}}}{{\rm{e}}}$ As $n \to \infty $, we get $\dfrac{1}{n} \to 0$. Therefore, $\mathop {\lim }\limits_{n \to \infty } {{\rm{e}}^{1/n}} = 1$, and $\mathop {\lim }\limits_{n \to \infty } {\left( {n + 1} \right)^{\dfrac{1}{n}}} = 1$. It follows that $\dfrac{1}{{\rm{e}}} \le \mathop {\lim }\limits_{n \to \infty } \dfrac{{\sqrt[n]{{n!}}}}{n} \le \dfrac{1}{{\rm{e}}}$ By the Squeezing Theorem for Sequences (Theorem 9.1.5), we obtain $\mathop {\lim }\limits_{n \to + \infty } \dfrac{{\sqrt[n]{{n!}}}}{n} = \dfrac{1}{{\rm{e}}}$
Update this answer!

You can help us out by revising, improving and updating this answer.

Update this answer

After you claim an answer you’ll have 24 hours to send in a draft. An editor will review the submission and either publish your submission or provide feedback.